Existence of a limiting sum of random variables











up vote
1
down vote

favorite
1












Consider that $X_i$‘s are independent exponentially distributed random variables with mean $1/i$ (and thus variance $1/i^2$).Then the sum of them seems to converge to a “random variable” with finite variance but unbounded mean. What’s the problem here? Why does not such random variable exist?
Thanks










share|cite|improve this question




















  • 1




    By the same reasoning the "random variable" sequence ${Y_i}_{i=1}^{infty}$ defined by $Y_i=i$ for all $i$ (with prob 1) has partial sums $sum_{i=1}^n Y_i$ with zero variance but mean that goes to infinity. But there is no mystery in this. We certainly do not say that $sum_{i=1}^n i$ "converges" in any sense (it diverges to $infty$). [I put "random variable" in quotes simply because the ${Y_i}$ sequence is deterministic in this case.]
    – Michael
    Nov 14 at 22:49

















up vote
1
down vote

favorite
1












Consider that $X_i$‘s are independent exponentially distributed random variables with mean $1/i$ (and thus variance $1/i^2$).Then the sum of them seems to converge to a “random variable” with finite variance but unbounded mean. What’s the problem here? Why does not such random variable exist?
Thanks










share|cite|improve this question




















  • 1




    By the same reasoning the "random variable" sequence ${Y_i}_{i=1}^{infty}$ defined by $Y_i=i$ for all $i$ (with prob 1) has partial sums $sum_{i=1}^n Y_i$ with zero variance but mean that goes to infinity. But there is no mystery in this. We certainly do not say that $sum_{i=1}^n i$ "converges" in any sense (it diverges to $infty$). [I put "random variable" in quotes simply because the ${Y_i}$ sequence is deterministic in this case.]
    – Michael
    Nov 14 at 22:49















up vote
1
down vote

favorite
1









up vote
1
down vote

favorite
1






1





Consider that $X_i$‘s are independent exponentially distributed random variables with mean $1/i$ (and thus variance $1/i^2$).Then the sum of them seems to converge to a “random variable” with finite variance but unbounded mean. What’s the problem here? Why does not such random variable exist?
Thanks










share|cite|improve this question















Consider that $X_i$‘s are independent exponentially distributed random variables with mean $1/i$ (and thus variance $1/i^2$).Then the sum of them seems to converge to a “random variable” with finite variance but unbounded mean. What’s the problem here? Why does not such random variable exist?
Thanks







probability statistics random-variables






share|cite|improve this question















share|cite|improve this question













share|cite|improve this question




share|cite|improve this question








edited Nov 15 at 3:04

























asked Nov 14 at 22:35









user49229

144




144








  • 1




    By the same reasoning the "random variable" sequence ${Y_i}_{i=1}^{infty}$ defined by $Y_i=i$ for all $i$ (with prob 1) has partial sums $sum_{i=1}^n Y_i$ with zero variance but mean that goes to infinity. But there is no mystery in this. We certainly do not say that $sum_{i=1}^n i$ "converges" in any sense (it diverges to $infty$). [I put "random variable" in quotes simply because the ${Y_i}$ sequence is deterministic in this case.]
    – Michael
    Nov 14 at 22:49
















  • 1




    By the same reasoning the "random variable" sequence ${Y_i}_{i=1}^{infty}$ defined by $Y_i=i$ for all $i$ (with prob 1) has partial sums $sum_{i=1}^n Y_i$ with zero variance but mean that goes to infinity. But there is no mystery in this. We certainly do not say that $sum_{i=1}^n i$ "converges" in any sense (it diverges to $infty$). [I put "random variable" in quotes simply because the ${Y_i}$ sequence is deterministic in this case.]
    – Michael
    Nov 14 at 22:49










1




1




By the same reasoning the "random variable" sequence ${Y_i}_{i=1}^{infty}$ defined by $Y_i=i$ for all $i$ (with prob 1) has partial sums $sum_{i=1}^n Y_i$ with zero variance but mean that goes to infinity. But there is no mystery in this. We certainly do not say that $sum_{i=1}^n i$ "converges" in any sense (it diverges to $infty$). [I put "random variable" in quotes simply because the ${Y_i}$ sequence is deterministic in this case.]
– Michael
Nov 14 at 22:49






By the same reasoning the "random variable" sequence ${Y_i}_{i=1}^{infty}$ defined by $Y_i=i$ for all $i$ (with prob 1) has partial sums $sum_{i=1}^n Y_i$ with zero variance but mean that goes to infinity. But there is no mystery in this. We certainly do not say that $sum_{i=1}^n i$ "converges" in any sense (it diverges to $infty$). [I put "random variable" in quotes simply because the ${Y_i}$ sequence is deterministic in this case.]
– Michael
Nov 14 at 22:49












1 Answer
1






active

oldest

votes

















up vote
1
down vote













Since $X_i$'s are positive $sum_{i=1}^{n} X_i$ converges (to a possibly infinte ) sum $X$ and Monotone Convergence Theorem gives $EX=sum_{i=1}^{infty} frac 1 i =infty$. There is no way a random variable with infinite mean can have finite variance. It is not even true that $X <infty$ almost surely. [This can be shown using Laplace transforms].






share|cite|improve this answer























    Your Answer





    StackExchange.ifUsing("editor", function () {
    return StackExchange.using("mathjaxEditing", function () {
    StackExchange.MarkdownEditor.creationCallbacks.add(function (editor, postfix) {
    StackExchange.mathjaxEditing.prepareWmdForMathJax(editor, postfix, [["$", "$"], ["\\(","\\)"]]);
    });
    });
    }, "mathjax-editing");

    StackExchange.ready(function() {
    var channelOptions = {
    tags: "".split(" "),
    id: "69"
    };
    initTagRenderer("".split(" "), "".split(" "), channelOptions);

    StackExchange.using("externalEditor", function() {
    // Have to fire editor after snippets, if snippets enabled
    if (StackExchange.settings.snippets.snippetsEnabled) {
    StackExchange.using("snippets", function() {
    createEditor();
    });
    }
    else {
    createEditor();
    }
    });

    function createEditor() {
    StackExchange.prepareEditor({
    heartbeatType: 'answer',
    convertImagesToLinks: true,
    noModals: true,
    showLowRepImageUploadWarning: true,
    reputationToPostImages: 10,
    bindNavPrevention: true,
    postfix: "",
    imageUploader: {
    brandingHtml: "Powered by u003ca class="icon-imgur-white" href="https://imgur.com/"u003eu003c/au003e",
    contentPolicyHtml: "User contributions licensed under u003ca href="https://creativecommons.org/licenses/by-sa/3.0/"u003ecc by-sa 3.0 with attribution requiredu003c/au003e u003ca href="https://stackoverflow.com/legal/content-policy"u003e(content policy)u003c/au003e",
    allowUrls: true
    },
    noCode: true, onDemand: true,
    discardSelector: ".discard-answer"
    ,immediatelyShowMarkdownHelp:true
    });


    }
    });














     

    draft saved


    draft discarded


















    StackExchange.ready(
    function () {
    StackExchange.openid.initPostLogin('.new-post-login', 'https%3a%2f%2fmath.stackexchange.com%2fquestions%2f2998915%2fexistence-of-a-limiting-sum-of-random-variables%23new-answer', 'question_page');
    }
    );

    Post as a guest















    Required, but never shown

























    1 Answer
    1






    active

    oldest

    votes








    1 Answer
    1






    active

    oldest

    votes









    active

    oldest

    votes






    active

    oldest

    votes








    up vote
    1
    down vote













    Since $X_i$'s are positive $sum_{i=1}^{n} X_i$ converges (to a possibly infinte ) sum $X$ and Monotone Convergence Theorem gives $EX=sum_{i=1}^{infty} frac 1 i =infty$. There is no way a random variable with infinite mean can have finite variance. It is not even true that $X <infty$ almost surely. [This can be shown using Laplace transforms].






    share|cite|improve this answer



























      up vote
      1
      down vote













      Since $X_i$'s are positive $sum_{i=1}^{n} X_i$ converges (to a possibly infinte ) sum $X$ and Monotone Convergence Theorem gives $EX=sum_{i=1}^{infty} frac 1 i =infty$. There is no way a random variable with infinite mean can have finite variance. It is not even true that $X <infty$ almost surely. [This can be shown using Laplace transforms].






      share|cite|improve this answer

























        up vote
        1
        down vote










        up vote
        1
        down vote









        Since $X_i$'s are positive $sum_{i=1}^{n} X_i$ converges (to a possibly infinte ) sum $X$ and Monotone Convergence Theorem gives $EX=sum_{i=1}^{infty} frac 1 i =infty$. There is no way a random variable with infinite mean can have finite variance. It is not even true that $X <infty$ almost surely. [This can be shown using Laplace transforms].






        share|cite|improve this answer














        Since $X_i$'s are positive $sum_{i=1}^{n} X_i$ converges (to a possibly infinte ) sum $X$ and Monotone Convergence Theorem gives $EX=sum_{i=1}^{infty} frac 1 i =infty$. There is no way a random variable with infinite mean can have finite variance. It is not even true that $X <infty$ almost surely. [This can be shown using Laplace transforms].







        share|cite|improve this answer














        share|cite|improve this answer



        share|cite|improve this answer








        edited Nov 14 at 23:35

























        answered Nov 14 at 23:30









        Kavi Rama Murthy

        41.4k31751




        41.4k31751






























             

            draft saved


            draft discarded



















































             


            draft saved


            draft discarded














            StackExchange.ready(
            function () {
            StackExchange.openid.initPostLogin('.new-post-login', 'https%3a%2f%2fmath.stackexchange.com%2fquestions%2f2998915%2fexistence-of-a-limiting-sum-of-random-variables%23new-answer', 'question_page');
            }
            );

            Post as a guest















            Required, but never shown





















































            Required, but never shown














            Required, but never shown












            Required, but never shown







            Required, but never shown

































            Required, but never shown














            Required, but never shown












            Required, but never shown







            Required, but never shown







            Popular posts from this blog

            Plaza Victoria

            In PowerPoint, is there a keyboard shortcut for bulleted / numbered list?

            How to put 3 figures in Latex with 2 figures side by side and 1 below these side by side images but in...